• ベストアンサー

単射の総数

|A|=m,|B|=n(m<_n)のときの単射f:A→Bの総数を求めよ この考え方もわかりません どなたか教えてください

質問者が選んだベストアンサー

  • ベストアンサー
  • hitomura
  • ベストアンサー率48% (325/664)
回答No.3

>どうしてf(a(m))の取りうる値はn-m+1通り なのですか? 単写、という言葉の意味は分かりますよね?念のため書きますが  a≠bならばf(a)≠f(b) という意味です。 f(a(1))のとりうる値の種類はBの要素数のn個です。 f(a(2))のとりうる値の種類は集合Bからf(a(1))の値として選んだ値を除いた集合の個数であるn-1個です。 f(a(3))のとりうる値の種類は「集合Bからf(a(1))の値として選んだ値を除いた集合」から(a(2))の値として選んだ値を除いた集合の個数である(n-1)-1=n-2個です。 f(a(3))のとりうる値の種類は… ということをa(m)まで繰り返していけば、a(m)のとりうる値の種類がn-m+1個になることが分かります。 なお、上の「…」で省略した部分が分からない、といわれても、私にはそこをやっていく根気はありません。

その他の回答 (2)

  • starflora
  • ベストアンサー率61% (647/1050)
回答No.2

    これは単射の問題なので、そのまま考えます。m<=nというのが条件だと理解します。     Bの要素のなかで、n-m個が、写像とは関係のない、対応要素のない要素になります。従って、これを排除して考えるのがよいことになります。こういうn-m要素を外した、Bの部分集合の異なる可能性の数は、nから任意にn-m個の要素を選ぶ組み合わせです。これは、nCn-mです。具体的には、n(n-1)(n-2)……(n-m+1)/m!=nPn-m/m!=rです。(rは適当な文字です)。     要素が違うr個の集合があるということになり、濃度は、Aと同じです。     このr個の集合のそれぞれに対し,Aからの単射があることになります。その単射の組み合わせ数は、m!となります。     先のr個の集合は、みな異なる集合です。何故なら、どれも、最低、一個の要素が食い違っているからです。つまり、仮にBから、{a,b}要素と{a,c}要素を除いた二つの集合を考えると、先にはcがあるがbがなく、後は、bはあるがcがないという風に違った集合です。     r個の集合をBrと表現すると(r=1,2,3……)、A→Brの単射は、いかなる組み合わせを造っても、同じものはないということになります。何故なら、Brは、rが違えば、それぞれ別の集合だからです。     一般に別個のm個の要素からなる集合から、同じように、別個のm個の集合への単射の数は、すでに上でも述べたように、m!です。従って、Aに対するBrの数をかければ、これが、問題の答えです。すなわち、(nCn-m)X(m!)です。ところで、nCn-m というのは、nPn-m/m!です。これにm!をかけるのですから、答えは、nPn-m です。     回答: nP(n-m)  

  • hitomura
  • ベストアンサー率48% (325/664)
回答No.1

http://oshiete1.goo.ne.jp/kotaeru.php3?q=209227 のほうでも書きましたが、|A|はAの要素数という意味でしょうか? ここではそうだと解釈します。 A={a(1),a(2),…,a(m)}とします。すると、fは単車なので、  f(a(1))の取りうる値はn通り、  f(a(2))の取りうる値はn-1通り、   …  f(a(m))の取りうる値はn-m+1通り、 となります。これを掛け合わせると、  n*(n-1)*…*(n-m+1)={n*(n-1)*…*1}/{(n-m)*(n-m-1)*…*1}           =n!/(n-m)!           =nP(n-m) となります(ただし、nP(n-m)のnおよびn-mは下添字)。

mahiro19
質問者

補足

どうしてf(a(m))の取りうる値はn-m+1通り なのですか?

関連するQ&A

  • 単射と全射

    Bをn行m列複素行列とし、Bを表現行列とするf;m元複素数→n次元複素数への線形写像とする。また、転置Bはm行n列複素行列でありg;n次元複素数→m次元複素数への線形写像であるとする。 このときfが単射ならば、gは全射であることを示せ。

  • 単射の証明

    「A→Bへの写像fに対して、 fが単射⇔g・f=idA となるBからAへの写像gが存在することを証明」 という問題なのですが、たぶん「 |f^-1(b)|≦1 」を使うと思うのですが... そこからどうすればいいか教えていただけないでしょうか。

  • 写像に関する問題で単射、全射、全単射を選ぶ問題についての質問です

    大学の問題で、 関数f,g:N→Nを以下のように定義する。 f(n) = 3n, g(n) = [n/3]+1     ※[ ]は床関数を表す fとgの合成gfが満たす性質を選べ。 (A)単射でも全射でもない(B)単射だが全射ではない (C)全射だが単射ではない(D)全単射である という問題なのですが、gfが1となる元が存在しないので(B)の単射だが全射ではないと思うのですが、回答を見たら(D)の全単射でした。なぜ全射になるのか分らないのですが、教えていただけないでしょうか。 よろしくお願いします。

  • 連続単射

    いかにも大学教養レベルの位相の問題なんですが、少し混乱してしまっています。どなたかご教示いただけたらと思います。 R^n→R^mへの連続単射fがあったとします。疑問点は三つです。 (i)m≧nか?像f(R^n)に制限すれば連続全単射になります。したがって局所コンパクトからハウスドルフへの連続全単射が存在することになって、局所同相ですが、m<nならそれは位相的にあり得ないように思います。この論証は正しいですか。 (ii)上のことが正しいとして、m≧nを仮定します。一般にfは閉写像ではないと思います。たとえばm=n=1ならf(x)=e^xとおけば、閉集合Rを開集合(0,∞)にうつすからです。一般のm,nではこれも少し自信がありません。閉写像にならない反例は常にあげられるでしょうか。 (iii)またm>nなら単純な埋め込みf(x)→(x,0)(残りの成分を0とおく)、を考えれば、開写像でないのは明らかですが、ではn=mのときはどうか。これがいちばん知りたいことですが、たとえばn=m=1のとき、R上の連続単射を考えていることになって、fは狭義単調。したがって逆もまたそうであって、像に制限すれば同相です。特にR上の単調関数は開区間を開区間にうつします。問題はn=m>1のときで、これもやはり開写像になるのでしょうか。局所同相がきちんと言えると示せなくもないような気がするのですが、困っています。

  • 写像の単射全射のところの関係式に関する証明について

    写像の単射全射のところの証明がわからないので、ご教授ください。 集合AからBへの写像をfとし、a∈A,P⊂A,b∈B,Q⊂Bとする。 1.fが単射のとき、a∈P ⇒ f(a)∈f(P)の逆が成り立つことの証明 2.fが単射のとき、P1⊂P2 ⇒ f(P1)⊂f(P2)の逆が成り立つことの証明 3.fが単射のとき、f(A-P) ⊃ f(A) - f(P) の逆が成り立つことの証明 4.fが単射のとき、f^(-1)(f(P)) = Pの証明 5.fが全射のとき、∃a'∈f^(-1)(Q), b=f(a') ⇒ b∈Qの逆が成り立つことの証明 6.fが全射のとき、Q1⊂Q2 ⇒ f^(-1)(Q1)⊂f^(-1)(Q2)の逆が成り立つことの証明 7.fが全射のとき、f(f^(-1)(Q)) = Qの証明 以上の7問です。 何個かだけでも構いませんので、回答して頂ければ嬉しいです。 また、はじめての質問ですので、ご迷惑をおかけするかもしれませんが、よろしくお願いいたします。

  • モンモール問題、完全順列、攪乱順列の拡張

    モンモール問題、完全順列、攪乱順列で検索するといろいろな言い回しがあります。 1,2,3,・・・,n の数を並び替えたとき、先頭から数えた順番と数が一致するものが1つもない並べ方 n人がプレゼントをもちよって、バラバラに交換したとき、1人も自分自身の用意したプレゼントをもらわない方法 写像f:{1,2,…,n}→{1,2,…,n}ただし、単射かつ∀i∈{1,2,…,n},f(i)≠i の総数 これらの場合の数は、n!Σ[k=0,n]{(-1)^k}/k!であることはよく知られています。 そこで、拡張として次の総数を考えるとどうなるのでしょうか? n≦mとする。 写像f:{1,2,…,n}→{1,2,…,m}ただし、単射かつ∀i∈{1,2,…,n},f(i)≠i の総数 たとえば、n=3,m=4のとき、 (f(1),f(2),f(3))=(2,1,4),(2,3,1),(2,3,4),(3,1,2),(3,1,4),(3,4,1),(3,4,2),(4,1,2),(4,3,1),(4,3,2)

  • 単射 全射 全単射 について教えてください

    タイトルの通り、単射 全射 全単射についていまいち納得できないので教えてください。 今、手元に問題が5つあるのですが 自然数、整数、実数全体の集合をそれぞれN,Z,Rとする。 (1)f:Z→N f(x)=x2(二乗) (2)f:R→R f(x)=2x(x乗) (3)f:R→R f(x)=sinx (4)f:Z→R f(x)=x3(三乗) (5)f:R→R f(x)=2x+1 例えば、(1)であれば  Zが1のとき、Nは1、Zが2のとき、Nは4という風にZが決定すればNはただひとつ必ず決まるから単射。 でも、Zが2のときは、Zは1とも-1ともいえるので全射ではない、ということなのでしょうか。 全単射、というのはそうするとどういった状態を言うのでしょうか・・・ それぞれの問題も全くちんぷんかんぷんです。 どうか教えてください。

  • 単射と全射について

    写像、単射、全射についての質問です。 これらのイメージがいまいちつかめません。 定義とか証明とかいったことが知りたいのでなく、 具体的な問題を解くための理解を得たいと思っています。 具体的な問題を挙げてみると、いまA={a,b,c,}とすると AからAへの写像の数は27になるそうですが、 これはaについて3通りあって、bについても3通りあって、cについても3通りあるから 3×3×3=27という考え方であっているでしょうか? 次に、AからAへの単射の数、全射の数はそれぞれ6通りあるそうですが、 これはどういう考え方なのでしょうか?おそらく3!という計算だと思うのですが、 なぜそのような計算をするかがわかりません。 単射については、行き先の値がダブってはいけないということなのでしょうか? 拙い日本語で申し訳ないのですが、 補足等必要ならいたしますのでどなたか詳しい方は教えてください。よろしくお願いします。

  • 単射

    線形空間Vは部分空間W1,W2の直和になっている。VからV'への線形写像fが単射ならImf=f(V) (+) f(W)となる これをしめしたいです。 線形空間VはW1,W2の直和であるため、 dim V=dimW1 + dimW2というのは解ります ここからどうしたら良いでしょうか? この命題がイメージ的に成立することは解ります。 ただ何故単射でなくてはいけないのでしょう?

  • 集合の問題で分からない問題があります。

    集合の問題で分からない問題があります。 どれだけ考えても証明の仕方がわからないので、教えていただけると助かります。 問.(M×N)\(A×B)=((M\A)×N)∪(M×(N\B))を示しなさい(「\」は差集合) 問.f:X→Yに対して、fが単射のとき、A、B⊆Xに対して f(A∩B)=f(A)∩f(B) が成り立つことを示しなさい。